I need help asap it’s due midnight

Answers

Answer 1

Hey buddy, I am there to help!

360 = x + x - 44 + 90 + 90

360 = 2x + 224

2x = 360 - 224

2x = 36

x = 36/2

x = 18

Plz mark my answer brainliest

Answer 2

Answer:

112

Step-by-step explanation:

[tex]x \degree + (x - 44) \degree = 180 \degree \\ (2x - 44) \degree = 180 \degree \\ 2x - 44 = 180 \\ 2x = 180 + 44 \\ 2x = 224 \\ x = \frac{224}{2} \\ x = 112 \\ (x - 44) = 112 - 44 = 68[/tex]


Related Questions

What is the value of x?



Enter your answer in the box.

Answers

Answer:

x=12

Step-by-step explanation:

Both Angles together have a 180 degrees, set your formula up as

180=(10x-20)+(6x+8)

180=10x+6x-20+8

180+20-8=16x

192=16x

192/16=x

12=x

Answer:

x = 12

Step-by-step explanation:

The two angles form a straight line so they add to 180

10x-20 + 6x+8 = 180

Combine like terms

16x -12 = 180

Add 12 to each side

16x-12+12=180+12

16x = 192

Divide each side by 16

16x/16 =192/16

x = 12

Jim orders prints from a website. The site charges him $6.95 a month and $0.04 for each print he orders.
Enter an equation that can be used to find the number of prints, P, Jim ordered last month if the website charged
him $17.79.
Enter the number of prints Jim ordered last month.

Answers

Answer:

271 prints.

Step-by-step explanation:

So to answer the question, we need to first write a formula to find the answer.

We know that the starting price per month is 6.95 dollars.

The full price is 17.79 dollars.

So we can start this off by writing:

17.79=6.95

Now, every time Jim prints, he charges 0.04 dollars. This will add to the orginal cost, which is 6.95. And remember this isnt a single payment, but a payment that varies depending on the amount of paper, p. So we can write this as:

17.79=6.95+0.04p

6.95 wont ever change, hense why it doesnt have a variable. 0.04 however, varies depending on the amount of paper, and thats why we included the variable p.

These two values combined make 17.79, hense why it is on the opposite side of the equals sign.

So, now we need to find how many papers, p, were printed.

To find this, lets just take our equation, and solve for P like an ordinary problem:

17.79=6.95+0.04p

So first off, lets subtract the 6.95 from both sides, because that iscolates the 0.04p, which contains the variable.

Now we have:

10.84=0.04p

Now, we could divide both sides by 0.04.

But remember, the whole point of doing this is to make 0.04p into 1p.

So another way of doing this is multiplying the 0.04 by 25, since 0.04*25=1.

So lets multiply both sides by 25:

10.84*25 is 271.

Since 10*25 is 250

And

.84*25 is 21.

So now we know:

271=1p

So 271 is your answer!

Hope this helps!

The equation that can be used to find the number of prints, p, is $17.79 = $6.95 + $0.04p and Jim ordered 271 prints last month.

What is an equation?

An equation is an expression that shows the relationship between two or more numbers and variables.

We can write an equation to represent the total cost that Jim was charged by the website for last month's prints, p to represent the number of prints he ordered:

Total cost = Monthly fee + Cost per print x Number of prints

Using the values given in the problem:

$17.79 = $6.95 + $0.04p

To solve for p, one side of the equation. First, we can subtract $6.95 from both sides:

$10.84 = $0.04p

Then, we can divide both sides by $0.04:

p = 271

To learn more about equation click on;

brainly.com/question/31112129

#SPJ2

What is the value of a?
A)168
B)108
C)54
D)30

Answers

D should be the correct answer

Eliana travels at an average speed of 50 mph for 60 miles. Without stopping, Eliana then travels 130 miles in 2.5 hours. Find her average speed for the entire journey to 2 dp.

Answers

Answer:

102 mph

Step-by-step explanation:

Eliana travels at an average speed of 50 mph for 60 miles.

Without stopping, Eliana then travels 130 miles in 2.5 hours.

We calculate the speed here

Speed = Distance in miles /Time in hours

Speed = 130 miles/2.5 hours

= 52 miles per hour

Hence, her average speed for the entire journey is calculated as:

50 mph + 52 mph

= 102 mph

Solve the equation
(0)
36
(6)
O A * = 8
O B x=7
0 CX=512
0 D
3

Answers

Answer

A.x=8

1/3x-2/3x=2

x-2=6

x=6x2

x=8

I NEED HELP ASAP!What is the area of the triangle shown below? 10 10 12​

Answers

Answer:

[tex]h^2+6^2=10^2[/tex]

[tex]h^2+36=100[/tex]

[tex]h^2=100-36[/tex]

[tex]h^2=64[/tex]

[tex]h=8[/tex]

[tex]A=1/2 ~BH[/tex]

[tex]A=1/2(12)(8)[/tex]

[tex]A=48 ~units^2[/tex]

-----------------------

hope it helps...

have a great day!!

Answer:  48 square units

=============================================================

Explanation:

Draw a vertical line to cut the triangle into two equal parts. You'll get two right triangles.

The horizontal leg of one of those smaller triangles is 12/2 = 6 units. The hypotenuse is 10 units. Through the pythagorean theorem, you should find the vertical leg is 8 units.

We have a 6-8-10 right triangle.

The area of this smaller triangle is base*height/2 = 6*8/2 = 48/2 = 24 square units.

So the area of the larger triangle is 2*24 = 48 square units.

Which number line represents the solution set for the inequality 3(8 - 4x) < 6(x - 5)?
+
4
-5
-4
-3
-2
-1 0
1 2 3
5
+
+
-5
+ + +
-2 -1 0
+
2 3 4 5
-4
-3
1
út
+
+
+
-2 -1 0
-4
+
3
-5
-3
+
4
1
2
5
-5
-4
-3
-2 -1 0
1
2 3
4 5
Zoom in to see number line clearly

Answers

-3 greater than x

so I guess it should be the THIRD ONE and sorry if my answer is wrong but i tried my best so enjoy!

the graph of the function f(x) is shown

Answers

Answer:

where is the graph

and what is your question

could you explain

Find q
(7q-46)
(3q+6)

Answers

Answer:

[tex]q = 22[/tex]

Step-by-step explanation:

The angle measure of a straight line is (180) degrees As given in the picture, line (AC) is a straight line, thus its angle measure is (180) degrees. Angles (ABD) and (DBC) make up line (AC). Therefore, one can conclude that the sum of angles (ABD) and (DBC) is (180) degrees. Form an equation, simplify and use inverse operations to solve for the unknown parameter (q).

[tex](<ABD) + (<DBC) = (AC)[/tex]

Substitute for the angle measures,

[tex](7q-46)+(3q+6)=180[/tex]

Simplify,

[tex]10q-40=180[/tex]

Inverse operations,

[tex]10q-40=180\\\\10q = 220\\\\q = 22[/tex]

Answer:

need pts

Step-by-step explanation:

The following table represents the highest educational attainment of all adult
residents in a certain town. If a resident who is aged 40-49 is chosen at random, what
is the probability that they have completed a master's degree? Round your answer to
the nearest thousandth.

Answers

612/15564 = .04 lmk if it is correct

Answer:

≈0.576

Step-by-step explanation:

Age under 40: 871+870=1741

Total with master’s degree: 3023

Probability: 1741/3023

≈0.576

If x = -5 and y = 2x^2,
then y = ? *

Answers

Answer:

x = -5

y = 2×^2

y = 2 x (-5 x -5)

= 2 x 25

y = 50

A $20, 000 brand new car depreciate its value every year following the pattern, $16,000, $12,800, $10,240. How much will the car be after 5 years?

A.$10, 000
B.$8,192
C.$6553.6
D.$6000

Answers

Answer:

Step-by-step explanation:

Use the info given in the exponential equation to find the value of b, the rate of decay.

[tex]v(t)=a(b)^t[/tex] where v(t) is the value of the car after a certain number of years, t, have gone by, a is the initial value, and b is the rate of decay. We have everything we need but b:

a = 20000

v(t) = 16000 after t = 1 year:

[tex]16000=20000(b)^1[/tex] so

b = .8  Taken in context, this means that the car depreciates 20% each year. Now we can solve the problem being asked of us, which is to find the value of the car after t = 5 years:

[tex]v(t)=20000(.8)^5[/tex] which simplifies down a bit to

v(t) = 20000(.32768) so

v(t) = 6553.60, choice C.

The ratio of sheep to cows in a field is 3:8. The ratio of cows to pigs in the same field is 5:6. What is the ratio of sheep to cows to pigs?

Answers

Answer:

3:5:6

Step-by-step explanation:

because there are 3 sheep 5 vows and 6 pigs

The area ratio of two similar soilds is 169:289 if the volume of the smaller solid is 689,858 cm what is the volume of the larger solid

Answers

Answer: [tex]1,542,682\ cm^3[/tex]

Step-by-step explanation:

Given

The ratio of two similar figures is 169:289

Suppose, their length ratio is x:y

[tex]\therefore \dfrac{x^2}{y^2}=\dfrac{169}{289}\\\\\Rightarrow \dfrac{x}{y}=\dfrac{13}{17}[/tex]

Similarly, cube of the ratio is equal to the volume ratio

[tex]\Rightarrow \left(\dfrac{13}{17}\right)^3=\dfrac{689,858}{V}\\\\\Rightarrow V=689,858\times \dfrac{17^3}{13^3}\\\\\Rightarrow V=314\times 17^3\\\\\Rightarrow V=1,542,682\ cm^3[/tex]

Thus, the volume of the larger solid is [tex]1,542,682\ cm^3[/tex]

Select the correct answer.
Simplify the expression so there is only one positive power for the base, -5.
–5? = -52
OA
1
59
O B.
-59
OC.
--55
OD.
55

Answers

Answer:

I think the answer is C.

Step-by-step explanation:

The law of exponents states that when you divide the same base you subtract the exponents.

hi please help will mark brainliest Charlie and Andie solve this problem in two different ways:

A family of 6 spent $74 at the movie theater.
They bought $26 worth of snacks and a ticket for each family member.
What is the price of each ticket if all tickets are the same price​

Answers

Answer:

8$

Step-by-step explanation:

since it was 26$ worth of snacks, first subtract 74 from 26

74-26= 48$ left

from this 48$, they bought tickets, therefore price for one ticket= 48/6=8$

therefore each ticket cost 8$

Answer:

hope this helps!

Step-by-step explanation:

subtract/divide/a variable

credit: edgarandres9741

source: https://brainly.com/question/11711576

Triangles A B C and R M Q are shown. Angles C A B and M R Q are 29 degrees. Angles A B C and R M Q are 116 degrees.
What additional information could be used to prove that the triangles are congruent using AAS? Select two options.

AngleC ≅ AngleQ
CB ≅ QM
AC = 3.9cm and RQ = 3.9cm
mAngleC = 35° and mAngleQ = 35°
AB = 2.5cm and MQ = 2.5cm

Answers

Answer:

Option 1: AC= 3.9cm and RQ=3.9cm

Option 2: CB = QM

Step-by-step explanation:

i think because you need a side

The additional information needed to prove both triangles are congruent by AAS is any of the following:

C. AC = 3.9cm and RQ = 3.9cm

E. BC = 2.5cm and MQ = 2.5cm

What is the AAS Theorem?

Based on the AAS theorem, two triangles are proven to be congruent if two angles and one non-included side in one triangle are congruent to corresponding two angles and one non-included side in the other triangle.

From the image of the triangles, we know that two pairs of the angles, 29° and 116°, are congruent to each other in the two triangles.

For us to prove that both triangles area congruent by the AAS theorem, we will need to know either of the following: C. AC = 3.9cm and RQ = 3.9cm or E. BC = 2.5cm and MQ = 2.5cm.

Learn more about the AAS Theorem on:

https://brainly.com/question/14635501

#SPJ6

?
Which is the graph of f(x)

Answers

Answer:

pls provide all the option

Answer:

Neither of ur options :(

Step-by-step explanation:

[tex]f(x) = 3(\frac{2}{3})^x[/tex]

y - intercept :

[tex]x = 0, f(x) = 3 \times (\frac{2}{3})^0 = 3[/tex]

Option 1 : y intercept is 6

option 2 : y intercept is 6

Figure A is a scale image of Figure B what is the value of x

Answers

Answer:

72

Step-by-step explanation:

20 got 4 added to it and the only number that gives you 24 when you multiply it is 4x6 and we can prove that to be right because the one before it is 4x5 and it gives us 20. Now that we know the multiplier is 5 and 6 we can ask ourselves, "What do I multiply by 5 that gives me 60?" And that is 12. So 12x6 is 72 for x.

Question 4 of 11
One way to find the equation of a line is to look at some points on it and try to
find the relationship between the coordinates.
A. True
B. False

Answers

This would be A True, hope this helps !

solve for q tysm:)))​

Answers

Question

Solve for q

-----------------

[tex](8q-5)^o+81=180^o[/tex]

[tex]8q-5+81=180[/tex]

[tex]8q+76=180[/tex]

[tex]8q=180-76[/tex]

[tex]8y=104[/tex]

[tex]q=13[/tex]

---------------

Answer[tex]q=13[/tex]-----------------------hope it helps...have a great day!!

If the value of "Y" varies directly with "X" and y=-8,when x=20,find "Y"if x=-4

Answers

Answer:

y is 8/5 when x = -4.

Step-by-step explanation:

If the value of "Y" varies directly with "X", we can write y = kx, where k is the constant of proportionality.

If "y=-8,when x=20," then we can find k:    -8 = 20k, or k = -2/5.

Then y = (-2/5)x.

If x = -4, then y = (-2/5)(-4) = 8/5

y is 8/5 when x = -4.

Joe's broker's fee schedule is given in the table. Joe's current portfolio is worth $50,000. He wants to purchase 50 shares of a company's
stock for a purchase price of $15 per share.
Joe will need to pay a commission of
. The total amount charged by the broker will be
Part one answers $7.50, $5.00,$1.50
Part two answers $17.50,$15.50,$15.00

Answers

Answer:

Joe Will need to pay a commission of [$7.50]. The Total amount Charged by the broker will be [$17.50]

Step-by-step explanation:

Joe will need to pay a commission of [tex]\$7.50[/tex] and Total amount charged by broker will be [tex]\$17.50[/tex].

What is commission ?

Commission is a form of money pay as remuneration for services rendered or products sold.

A.

We have,

Joe's current portfolio  [tex]=\$ 50,000[/tex]

Price of one stock [tex]=\$15[/tex]

So,

As his portfolio is less than [tex]\$100,000[/tex], he have to pay [tex]15\%[/tex] and [tex]\$10[/tex] trade fee,

So,

Price of [tex]50[/tex] shares [tex]=50*15=\$750[/tex]

Commission charge [tex]=1\%\ of\ 750[/tex]

                                  [tex]=\$7.5[/tex]

So, commission of [tex]\$7.5[/tex] will be paid.

B.

So,

Total amount charged by Broker [tex]=10+7.5=\$17.5[/tex]

Hence, we can say that Joe will need to pay a commission of [tex]\$7.50[/tex] and Total amount charged by broker will be [tex]\$17.50[/tex].  .

To know more about probability click here

https://brainly.com/question/20987196

#SPJ3

HELP PEASE I WILL MARK BRAINLIEST ASAP !!!

Find the measure of the arc.
с
А
146°
E
B
mÉFD = [?]°

HELP PEASE I WILL MARK BRAINLIEST ASAP !!!

Answers

The answer is 270 because there is a 90° angle and 146n

Perform (3x^3 – 2x^2 + 3x – 4) ÷ (x – 3) to find the value of the remainder.
Question 2 options:

A)
68
B)
88
C)
98
D)
78

Answers

I believe the answers is B

Answer:

a) 68

Step-by-step explanation:

using the remainder theorem.

3x^3 - 2x^2 + 3x - 4 ÷ x - 3

p(a) = 3x^3 - 2x^2 + 3x - 4

p(3) = 3(3)^3 - 2(3)^2 + 3(3) - 4

= 3(27) - 2(9) + 9 - 4

= 81 - 18 + 9 - 4

= 68

HELP NOW!!! What are all possible values of x?

Answers

14 because of the formation and the formula

Simplify. (-2)-³
8
1/8
-8
-1/8​

Answers

Answer:

[tex]-\frac{1}{8}[/tex]

Step-by-step explanation:

Assuming you mean

(- 2)⁻³

- 0.125

[tex]-\frac{1}{8}[/tex]

Answer:

-8

Step-by-step explanation:

see image below:)

The kitchen, dining room, and living room of a house will be painted, each with a different color. There are 7 colors to choose from. This means that there are _[blank]_ color arrangements possible.

Answers

Answer:

21

Step-by-step explanation:

Given that :

Number of rooms = 3

Variants of paint available = 7

Since the rooms will be painted with different colors :

Hence,

Number of possible paintings for kitchen = 7 (7 different paints)

This also applies to the other 2 rooms

Hence,

We have 3 * 7 = 21 color arrangements possible.

Peter works at a camera store. He is paid an hourly rate, plus 2 points
16% commission on everything he sells. One week, he was paid $515 for
working 20 hours and selling $1,500 worth of camera equipment. What is
his hourly rate? *
A) $12.00 per hour
B) $13.75 per hour
C) $21.63 per hour
D) $25.75 per hour

PLSS HELP

Answers

Answer:

fdd

yfddd

Step-by-step explanation:

dded fdd fdd fdd fddc rttyj hreef ggg

Find the value of x
A.)20
B.)55
C.)70
D.)38

Answers

Answer:

A.) 20

Step-by-step explanation:

These angles are complementary meaning they add up to 90°.

4x - 10 + x = 90

5x - 10 = 90

5x = 100

x = 20

Answer:

Hello! answer: x = 20

Step-by-step explanation:

This is a complementary angle meaning it will add up to 90 degrees so...

20 × 4 = 80 80 - 10 = 70

70 + 20 = 90 therefore x = 20

Other Questions
Match the definition to the term. 1. amplifies tone voice purity 2. accuracy of tone resonance 3. a solemn promise or pledge of loyalty trite 4. worn-out, overused plight 5. the state of being able to adapt to change flexibility 10 points:) please help i keep getting the wrong answer How many grams of AlCl3 are needed to completely react with 2.25 of NaOH? What is the solution to the equation -5p = 24-p?Op= -6O p= - 27OpO p= 4O p=2425 What does good exposition in a story do for readers Match each date to the correct description.1. autumnal equinox June 21 2. vernal equinox September 22 3. longest day in Northern Hemisphere December 22 4. longest day in Southern Hemisphere March 21 The area of a rectangular fountain is (x^2+ 13x + 22) ft. The width is (x + 2) ft.a. Find the length of the fountain. Find the area of the bedroom with the shape shown in the figure, in square feet. A) 67 ft2 B) 106 ft2 C) 506 ft2 D) 548 ft2 Which of the following is a disadvantage to using tables? Check all that apply.MULTIPLE CHOICENO LINKSA. Graphs are much easier to work with for all problems.B. They force you to do every problem twice, once with a table and once with an equation.c. It is often difficult to solve complicated problems with tables alone.D. They organize the data. At one college, the tuition for a full-time student is $6,000 per semester. It has been announced that the tuition will increase by 2 percent each year for the next five years. Design a program with a loop that displays the projected semester tuition amount for the next five years. The good boy behaved well. (Underline the adverb in the sentence) Geometry end of year review escape room: room A Convert to percentages. List answers starting from the top. *41% 1.8366/50 help me please really need it HELP FAST PLS !!!!!!!!!!!!!!!! Select the correct answer.What is the solution for x in the equation?-3x + 7x 8 = 34 + 9x 2A. B. C. D. BRAINIEST AND 10 POINTS Key Company acquires 60, 10%, 5 year, $1,000 Community bonds on January 1, 2012 for $61,250. This includes a brokerage commission of $1,250. The journal entry to record this investment includes a debit to what type of translation is this ( x , y ) ( y , - x ) Movement of Earth's crust along plate boundaries produces *A) FrontsB). tidesC) hurricanesD) earthquakes